FINChap 13

Lakukan tugas rumah & ujian kamu dengan baik sekarang menggunakan Quizwiz!

The systematic risk of the market is measured by: A. a beta of 1.0. B. a beta of 0.0. C. a standard deviation of 1.0. D. a standard deviation of 0.0. E. a variance of 1.0.

A. a beta of 1.0

Which one of the following measures the amount of systematic risk present in a particular risky asset relative to the systematic risk present in an average risky asset? A. beta B. reward-to-risk ratio C. risk ratio D. standard deviation E. price-earnings ratio

A. beta

The market rate of return is 11 percent and the risk-free rate of return is 3 percent. Lexant stock has 3 percent less systematic risk than the market and has an actual return of 12 percent. This stock: A. is underpriced. B. is correctly priced. C. will plot below the security market line. D. will plot on the security market line. E. will plot to the right of the overall market on a security market line graph.

A. is underpriced

Standard deviation measures which type of risk? A. total B. nondiversifiable C. unsystematic D. systematic E. economic

A. total

How many diverse securities are required to eliminate the majority of the diversifiable risk from a portfolio? A. 5 B. 10 C. 25 D. 50 E. 75

B. 10

Which one of the following should earn the most risk premium based on CAPM? A. diversified portfolio with returns similar to the overall market B. stock with a beta of 1.38 C. stock with a beta of 0.74 D. U.S. Treasury bill E. portfolio with a beta of 1.01

B. stock with a beta of 1.38

The _____ tells us that the expected return on a risky asset depends only on that asset's nondiversifiable risk. A. efficient markets hypothesis B. systematic risk principle C. open markets theorem D. law of one price E. principle of diversification

B. systematic risk principle

Which one of the following statements related to risk is correct? A. The beta of a portfolio must increase when a stock with a high standard deviation is added to the portfolio. B. Every portfolio that contains 25 or more securities is free of unsystematic risk. C. The systematic risk of a portfolio can be effectively lowered by adding T-bills to the portfolio. D. Adding five additional stocks to a diversified portfolio will lower the portfolio's beta. E. Stocks that move in tandem with the overall market have zero betas.

C. The systematic risk of a portfolio can be effectively lowered by adding T-bills to the portfolio.

A stock with an actual return that lies above the security market line has: A. more systematic risk than the overall market. B. more risk than that warranted by CAPM. C. a higher return than expected for the level of risk assumed. D. less systematic risk than the overall market. E. a return equivalent to the level of risk assumed.

C. a higher return than expected for the level of risk assumed.

Treynor Industries is investing in a new project. The minimum rate of return the firm requires on this project is referred to as the: A. average arithmetic return. B. expected return. C. market rate of return. D. internal rate of return. E. cost of capital.

E. cost of capital

Which one of the following is represented by the slope of the security market line? A. reward-to-risk ratio B. market standard deviation C. beta coefficient D. risk-free interest rate E. market risk premium

E. market risk premium

Which one of the following is the formula that explains the relationship between the expected return on a security and the level of that security's systematic risk? A. capital asset pricing model B. time value of money equation C. unsystematic risk equation D. market performance equation E. expected risk formula

A. capital asset pricing model

Which one of the following is least apt to reduce the unsystematic risk of a portfolio? A. reducing the number of stocks held in the portfolio B. adding bonds to a stock portfolio C. adding international securities into a portfolio of U.S. stocks D. adding U.S. Treasury bills to a risky portfolio E. adding technology stocks to a portfolio of industrial stocks

A. reducing the number of stocks held in the portfolio

Which one of the following statements is correct concerning unsystematic risk? A. An investor is rewarded for assuming unsystematic risk. B. Eliminating unsystematic risk is the responsibility of the individual investor. C. Unsystematic risk is rewarded when it exceeds the market level of unsystematic risk. D. Beta measures the level of unsystematic risk inherent in an individual security. E. Standard deviation is a measure of unsystematic risk.

B. Eliminating unsystematic risk is the responsibility of the individual investor.

Systematic risk is measured by: A. the mean. B. beta. C. the geometric average. D. the standard deviation. E. the arithmetic average.

B. beta

The capital asset pricing model (CAPM) assumes which of the following? I. a risk-free asset has no systematic risk. II. beta is a reliable estimate of total risk. III. the reward-to-risk ratio is constant. IV. the market rate of return can be approximated. A. I and III only B. II and IV only C. I, III, and IV only D. II, III, and IV only E. I, II, III, and IV

C. I, III, and IV only

The primary purpose of portfolio diversification is to: A. increase returns and risks. B. eliminate all risks. C. eliminate asset-specific risk. D. eliminate systematic risk. E. lower both returns and risks.

C. eliminate asset-specific risk

Which one of the following is a risk that applies to most securities? A. unsystematic B. diversifiable C. systematic D. asset-specific E. total

C. systematic

The _____ of a security divided by the beta of that security is equal to the slope of the security market line if the security is priced fairly. A. real return B. actual return C. nominal return D. risk premium E. expected return

D. risk premium

Which one of the following statements is correct? A. The unexpected return is always negative. B. The expected return minus the unexpected return is equal to the total return. C. Over time, the average return is equal to the unexpected return. D. The expected return includes the surprise portion of news announcements. E. Over time, the average unexpected return will be zero.

E. Over time, the average unexpected return will be zero.

Which one of the following indicates a portfolio is being effectively diversified? A. an increase in the portfolio beta B. a decrease in the portfolio beta C. an increase in the portfolio rate of return D. an increase in the portfolio standard deviation E. a decrease in the portfolio standard deviation

E. a decrease in the portfolio standard deviation

Which of the following statements concerning risk are correct? I. Nondiversifiable risk is measured by beta. II. The risk premium increases as diversifiable risk increases. III. Systematic risk is another name for nondiversifiable risk. IV. Diversifiable risks are market risks you cannot avoid. A. I and III only B. II and IV only C. I and II only D. III and IV only E. I, II, and III only

A. I and III only

Unsystematic risk: A. can be effectively eliminated by portfolio diversification. B. is compensated for by the risk premium. C. is measured by beta. D. is measured by standard deviation. E. is related to the overall economy.

A. can be effectively eliminated by portfolio diversification

Which one of the following is an example of systematic risk? A. investors panic causing security prices around the globe to fall precipitously B. a flood washes away a firm's warehouse C. a city imposes an additional one percent sales tax on all products D. a toymaker has to recall its top-selling toy E. corn prices increase due to increased demand for alternative fuels

A. investors panic causing security prices around the globe to fall precipitously

The intercept point of the security market line is the rate of return which corresponds to: A. the risk-free rate. B. the market rate. C. a return of zero. D. a return of 1.0 percent. E. the market risk premium.

A. the risk-free rate

Which one of the following statements is correct concerning a portfolio beta? A. Portfolio betas range between -1.0 and +1.0. B. A portfolio beta is a weighted average of the betas of the individual securities contained in the portfolio. C. A portfolio beta cannot be computed from the betas of the individual securities comprising the portfolio because some risk is eliminated via diversification. D. A portfolio of U.S. Treasury bills will have a beta of +1.0. E. The beta of a market portfolio is equal to zero.

B. A portfolio beta is a weighted average of the betas of the individual securities contained in the portfolio.

At a minimum, which of the following would you need to know to estimate the amount of additional reward you will receive for purchasing a risky asset instead of a risk-free asset? I. asset's standard deviation II. asset's beta III. risk-free rate of return IV. market risk premium A. I and III only B. II and IV only C. III and IV only D. I, III, and IV only E. I, II, III, and IV

B. II and IV only

According to CAPM, the amount of reward an investor receives for bearing the risk of an individual security depends upon the: A. amount of total risk assumed and the market risk premium. B. market risk premium and the amount of systematic risk inherent in the security. C. risk free rate, the market rate of return, and the standard deviation of the security. D. beta of the security and the market rate of return. E. standard deviation of the security and the risk-free rate of return.

B. market risk premium and the amount of systematic risk inherent in the security.

If a stock portfolio is well diversified, then the portfolio variance: A. will equal the variance of the most volatile stock in the portfolio. B. may be less than the variance of the least risky stock in the portfolio. C. must be equal to or greater than the variance of the least risky stock in the portfolio. D. will be a weighted average of the variances of the individual securities in the portfolio. E. will be an arithmetic average of the variances of the individual securities in the portfolio.

B. may be less than the variance of the least risky stock in the portfolio.

Suzie owns five different bonds valued at $36,000 and twelve different stocks valued at $82,500 total. Which one of the following terms most applies to Suzie's investments? A. index B. portfolio C. collection D. grouping E. risk-free

B. portfolio

Steve has invested in twelve different stocks that have a combined value today of $121,300. Fifteen percent of that total is invested in Wise Man Foods. The 15 percent is a measure of which one of the following? A. portfolio return B. portfolio weight C. degree of risk D. price-earnings ratio E. index value

B. portfolio weight

The excess return earned by an asset that has a beta of 1.34 over that earned by a risk-free asset is referred to as the: A. market risk premium. B. risk premium. C. systematic return. D. total return. E. real rate of return.

B. risk premium

The expected risk premium on a stock is equal to the expected return on the stock minus the: A. expected market rate of return. B. risk-free rate. C. inflation rate. D. standard deviation. E. variance.

B. risk-free rate

Which one of the following risks is irrelevant to a well-diversified investor? A. systematic risk B. unsystematic risk C. market risk D. nondiversifiable risk E. systematic portion of a surprise

B. unsystematic risk

Which one of the following events would be included in the expected return on Sussex stock? A. The chief financial officer of Sussex unexpectedly resigned. B. The labor union representing Sussex' employees unexpectedly called a strike. C. This morning, Sussex confirmed that its CEO is retiring at the end of the year as was anticipated. D. The price of Sussex stock suddenly declined in value because researchers accidentally discovered that one of the firm's products can be toxic to household pets. E. The board of directors made an unprecedented decision to give sizeable bonuses to the firm's internal auditors for their efforts in uncovering wasteful spending.

C. This morning, Sussex confirmed that its CEO is retiring at the end of the year as was anticipated.

Which one of the following is most directly affected by the level of systematic risk in a security? A. variance of the returns B. standard deviation of the returns C. expected rate of return D. risk-free rate E. market risk premium

C. expected rate of return

You own a stock that you think will produce a return of 11 percent in a good economy and 3 percent in a poor economy. Given the probabilities of each state of the economy occurring, you anticipate that your stock will earn 6.5 percent next year. Which one of the following terms applies to this 6.5 percent? A. arithmetic return B. historical return C. expected return D. geometric return E. required return

C. expected return

The expected rate of return on a stock portfolio is a weighted average where the weights are based on the: A. number of shares owned of each stock. B. market price per share of each stock. C. market value of the investment in each stock. D. original amount invested in each stock. E. cost per share of each stock held.

C. market value of the investment in each stock

Which one of the following will be constant for all securities if the market is efficient and securities are priced fairly? A. variance B. standard deviation C. reward-to-risk ratio D. beta E. risk premium

C. reward-to-risk ratio

Which of the following are examples of diversifiable risk? I. earthquake damages an entire town II. federal government imposes a $100 fee on all business entities III. employment taxes increase nationally IV. toymakers are required to improve their safety standards A. I and III only B. II and IV only C. II and III only D. I and IV only E. I, III, and IV only

D. I and IV only

The expected return on a portfolio: I. can never exceed the expected return of the best performing security in the portfolio. II. must be equal to or greater than the expected return of the worst performing security in the portfolio. III. is independent of the unsystematic risks of the individual securities held in the portfolio. IV. is independent of the allocation of the portfolio amongst individual securities. A. I and III only B. II and IV only C. I and II only D. I, II, and III only E. I, II, III, and IV

D. I, II, and III only

Which of the following statements are correct concerning diversifiable risks? I. Diversifiable risks can be essentially eliminated by investing in thirty unrelated securities. II. There is no reward for accepting diversifiable risks. III. Diversifiable risks are generally associated with an individual firm or industry. IV. Beta measures diversifiable risk. A. I and III only B. II and IV only C. I and IV only D. I, II and III only E. I, II, III, and IV

D. I, II, and III only

Which one of the following is the best example of a diversifiable risk? A. interest rates increase B. energy costs increase C. core inflation increases D. a firm's sales decrease E. taxes decrease

D. a firm's sales decrease

The expected return on a stock computed using economic probabilities is: A. guaranteed to equal the actual average return on the stock for the next five years. B. guaranteed to be the minimal rate of return on the stock over the next two years. C. guaranteed to equal the actual return for the immediate twelve month period. D. a mathematical expectation based on a weighted average and not an actual anticipated outcome. E. the actual return you should anticipate as long as the economic forecast remains constant.

D. a mathematical expectation based on a weighted average and not an actual anticipated outcome.

The reward-to-risk ratio for stock A is less than the reward-to-risk ratio of stock B. Stock A has a beta of 0.82 and stock B has a beta of 1.29. This information implies that: A. stock A is riskier than stock B and both stocks are fairly priced. B. stock A is less risky than stock B and both stocks are fairly priced. C. either stock A is underpriced or stock B is overpriced or both. D. either stock A is overpriced or stock B is underpriced or both. E. both stock A and stock B are correctly priced since stock A is riskier than stock B.

D. either stock A is overpriced or stock B is underpriced or both.

Which one of the following is a positively sloped linear function that is created when expected returns are graphed against security betas? A. reward-to-risk matrix B. portfolio weight graph C. normal distribution D. security market line E. market real returns

D. security market line

Total risk is measured by _____ and systematic risk is measured by _____. A. beta; alpha B. beta; standard deviation C. alpha; beta D. standard deviation; beta E. standard deviation; variance

D. standard deviation; beta

The market risk premium is computed by: A. adding the risk-free rate of return to the inflation rate. B. adding the risk-free rate of return to the market rate of return. C. subtracting the risk-free rate of return from the inflation rate. D. subtracting the risk-free rate of return from the market rate of return. E. multiplying the risk-free rate of return by a beta of 1.0.

D. subtracting the risk-free rate of return from the market rate of return.

A news flash just appeared that caused about a dozen stocks to suddenly drop in value by about 20 percent. What type of risk does this news flash represent? A. portfolio B. nondiversifiable C. market D. unsystematic E. total

D. unsystematic

The expected return on a stock given various states of the economy is equal to the: A. highest expected return given any economic state. B. arithmetic average of the returns for each economic state. C. summation of the individual expected rates of return. D. weighted average of the returns for each economic state. E. return for the economic state with the highest probability of occurrence.

D. weighted average of the returns for each economic state.

Which one of the following statements is correct concerning a portfolio of 20 securities with multiple states of the economy when both the securities and the economic states have unequal weights? A. Given the unequal weights of both the securities and the economic states, the standard deviation of the portfolio must equal that of the overall market. B. The weights of the individual securities have no effect on the expected return of a portfolio when multiple states of the economy are involved. C. Changing the probabilities of occurrence for the various economic states will not affect the expected standard deviation of the portfolio. D. The standard deviation of the portfolio will be greater than the highest standard deviation of any single security in the portfolio given that the individual securities are well diversified. E. Given both the unequal weights of the securities and the economic states, an investor might be able to create a portfolio that has an expected standard deviation of zero.

E. Given both the unequal weights of the securities and the economic states, an investor might be able to create a portfolio that has an expected standard deviation of zero.

The expected return on a portfolio considers which of the following factors? I. percentage of the portfolio invested in each individual security II. projected states of the economy III. the performance of each security given various economic states IV. probability of occurrence for each state of the economy A. I and III only B. II and IV only C. I, III, and IV only D. II, III, and IV only E. I, II, III, and IV

E. I, II, III, and IV

Which one of the following statements related to unexpected returns is correct? A. All announcements by a firm affect that firm's unexpected returns. B. Unexpected returns over time have a negative effect on the total return of a firm. C. Unexpected returns are relatively predictable in the short-term. D. Unexpected returns generally cause the actual return to vary significantly from the expected return over the long-term. E. Unexpected returns can be either positive or negative in the short term but tend to be zero over the long-term.

E. Unexpected returns can be either positive or negative in the short term but tend to be zero over the long-term.

The standard deviation of a portfolio: A. is a weighted average of the standard deviations of the individual securities held in the portfolio. B. can never be less than the standard deviation of the most risky security in the portfolio. C. must be equal to or greater than the lowest standard deviation of any single security held in the portfolio. D. is an arithmetic average of the standard deviations of the individual securities which comprise the portfolio. E. can be less than the standard deviation of the least risky security in the portfolio.

E. can be less than the standard deviation of the least risky security in the portfolio.

The standard deviation of a portfolio: A. is a measure of that portfolio's systematic risk. B. is a weighed average of the standard deviations of the individual securities held in that portfolio. C. measures the amount of diversifiable risk inherent in the portfolio. D. serves as the basis for computing the appropriate risk premium for that portfolio. E. can be less than the weighted average of the standard deviations of the individual securities held in that portfolio.

E. can be less than the weighted average of the standard deviations of the individual securities held in that portfolio.

Which one of the following is an example of unsystematic risk? A. income taxes are increased across the board B. a national sales tax is adopted C. inflation decreases at the national level D. an increased feeling of prosperity is felt around the globe E. consumer spending on entertainment decreased nationally

E. consumer spending on entertainment decreased nationally

The principle of diversification tells us that: A. concentrating an investment in two or three large stocks will eliminate all of the unsystematic risk. B. concentrating an investment in three companies all within the same industry will greatly reduce the systematic risk. C. spreading an investment across five diverse companies will not lower the total risk. D. spreading an investment across many diverse assets will eliminate all of the systematic risk. E. spreading an investment across many diverse assets will eliminate some of the total risk.

E. spreading an investment across many diverse assets will eliminate some of the total risk


Set pelajaran terkait

Introduction to Psychology Final

View Set

Chapter 8 - concepts - PED, YED, XED

View Set

Chapter 5 Review - Slopes & Equations

View Set

Combo with "Postpartum Care Part 2 from NCLEX-RN Maternal-Neonatal Nursing" and 8 others

View Set

Java OCP Part 6. Exception and Assertions

View Set

group decision making and problem solving chapter 11

View Set